13
$\begingroup$

I want to find some condition to construct a continuous finitely additive measure on the natural numbers, i.e. $f:P(\mathbb{N})\rightarrow [0,1]$ such that $f(\{n\})=0$, and $f$ is an additive measure.

I know in ZFC we can use an ultrafilter $U$ and define $f$ by $f(A)=1\Leftrightarrow A\in U$, but this is too trival.

How about ZF? or some other condition? like large cardinal.

$\endgroup$
5
  • $\begingroup$ How do you define whether or not $f$ is continuous? $\:$ $\endgroup$
    – user5810
    May 4, 2012 at 5:03
  • $\begingroup$ What do you mean by 'nature'? $\endgroup$
    – David Roberts
    May 4, 2012 at 5:47
  • $\begingroup$ @David: I think this means the set of natural numbers $\endgroup$
    – Yemon Choi
    May 4, 2012 at 6:15
  • 1
    $\begingroup$ Conversely for any such $f$, $f^{-1}(\{1\})$ is a non-principal ultrafilter. So it's just the same as asking about the "construction" of a non-principal ultrafilter on $\mathbf{N}$. $\endgroup$
    – YCor
    Feb 19, 2020 at 16:13
  • $\begingroup$ The range of $f$ is $[0,1]$, not $\{0,1\}$. Thus any set $A\subset\Bbb N$ such that $0<f(A)<1$ intersects any set of $f^{-1}(\{1\})$, but $A\not\in f^{-1}(\{1\})$. $\endgroup$ Dec 1, 2020 at 5:26

2 Answers 2

17
$\begingroup$

In ZF you can't prove that there is a finitely additive measure on $\mathcal P(\mathbb N)$ (giving all singletons measure 0 and $\mathbb N$ measure 1). The existence of such a measure gives you a subset of $\mathbb R$ without the Baire property, and such a set cannot be constructed without some help of the axiom of choice. In other words, there is no explicit "construction" of a non-trivial finitely additive measure.

Since you don't like the ultrafilter measure, here is one that is less trivial:

Instead of $\mathbb N$, we use the integers $\mathbb Z$ as the underlying set. For $n\in\mathbb N$ let $d_n(A)=\frac{|A\cap\{-n,\dots,n\}|}{2n+1}$. The sequence $(d_n(A))_{n\in\mathbb N}$ typically does not converge. We fix this using an ultrafilter $U$ on $\mathbb N$ as follows:

For each $A\subseteq\mathbb Z$ let $\mu(A)$ be the unique element of the set $\bigcap_{S\in U}\mbox{cl}(\{d_n(A):n\in S\})$. Here the closure is taken in $\mathbb R$. Now $\mu$ is a finitely additive measure on $\mathcal P(\mathbb Z)$ that is even translation invariant and hence very different from the ultrafilter measure that you find too trivial.

Notice that I have just proved the well known fact that the integers are an amenable group.

$\endgroup$
3
  • $\begingroup$ In ZF,The existence of such a measure gives you a subset of R without the Baire property,is this set ${X\subset N:f(X)=1}$? Can you tell me whether the set is nonmeasure? $\endgroup$ May 4, 2012 at 13:20
  • 3
    $\begingroup$ Jialiang, the collection of sets of full measure might be meager, but if you look instead at the collection of sets with measure at least $1/2$ you get a non-BP set (since complementation is a homeomorphism, it would have to be nonmeager, and it's not hard to show that any nonmeager BP subset of the power set of $\mathbb{N}$ contains three almost disjoint sets). You cannot rule out a measurable fapm on $\mathbb{N}$; in fact ZFC+MA proves that there are universally measurable fapms. $\endgroup$ May 4, 2012 at 14:08
  • $\begingroup$ Can you tell me why the full measure might be meager ? note,the measure is finitely additive measure ,giving all singletons measure 0 and N measure 1. I think I have a proof,fisrt the full measure set $F$ is a filter,by a theorem of Talagrand(Set theory on the Structure of the real line P206),it engough to prove:for every partition of $\omega$into finite sets$I_n$,there exists $X\in F$such that$\exists^\infty n X\cap I_n=\emptyset$.use a MAD fmmily and ccc of the measure we can get a measure zreo set have infinite interval. $\endgroup$ May 5, 2012 at 2:06
8
$\begingroup$

Eric van Douwen constructed a whole slew of finitely additive measures on the set of natural numbers, with various shifting and scaling properties in this paper. He discussed how much `choice' was needed for the constructions; often some version of the Hahn-Banach theorem would suffice.

$\endgroup$

Your Answer

By clicking “Post Your Answer”, you agree to our terms of service and acknowledge you have read our privacy policy.

Not the answer you're looking for? Browse other questions tagged or ask your own question.